I NEED HELP ASAP!!!!!!! Find 2 numbers that multiply to make -24 and add to make -10

Answers

Answer 1

Answer:

Step-by-step explanation:

-8*3= -24+14=-10

Answer 2

Answer:

-12 and 2.

Step-by-step explanation:

-12*2= -24,

-12+2=-10


Related Questions

Find the slope of the line passing through the points (8,-4) and (4, -8).

Answers

Answer:

1

Step-by-step explanation:

We can find the slope using

m= ( y2-y1)/(x2-x1)

   = ( -8 - -4)/( 4 - 8)

   =  ( -8 +4)/( 4 - 8)

   = -4 / -4

   = 1

Answer:

slope equals 1

Step-by-step explanation:

To do this you would need to do an equation that is [tex]\frac{y_{2}-y_{1} }{x_{2}-x_{1}}[/tex] so in this case -8 would be y2 and -4 would be y1 and 4 would be x2 and 8 would b e x1 so if you plug it into the equation we would get [tex]\frac{-8-(-4)}{4-8}[/tex] and if we simplify we get [tex]\frac{-4}{-4}[/tex] which simplifies to 1 so the slope would equal 1

Which pairs of angles are alternate exterior angles? select yes or no​

Answers

A - No

B - No

C - Yes

D - Yes

.

C and D are alternate exterior angles

A company manufacturing oil seals wants to establish X and R control charts on the process. There are 25 preliminary samples of size 5 on the internal diameter of the seal. The summary data (in mm) are as follows:
sigma^25_i = 1 X_t = 1, 253.75, sigma^25_i = 1 R_i = 14.08
(a) Find the control limits that should be used on the X and R control charts. For n = 5, A2 = 0.577, D4 = 2.114, D3 = 0
(b) Assume that the 25 preliminary samples plot in control on both charts. Estimate the process mean and standard deviation.

Answers

Answer:

A ) i) X control chart : upper limit = 50.475, lower limit = 49.825

    ii) R control chart : upper limit =  1.191, lower limit = 0

Step-by-step explanation:

A) Finding the control limits

grand sample mean = 1253.75 / 25 = 50.15

mean range = 14.08 / 25 = 0.5632

Based on  X control CHART

The upper control limit ( UCL ) =

grand sample mean + A2* mean range ) = 50.15 + 0.577(0.5632) = 50.475

The lower control limit (LCL)=

grand sample mean - A2 *  mean range = 50.15 - 0.577(0.5632) = 49.825

Based on  R control charts

The upper limit = D4 * mean range = 2.114 * 0.5632 = 1.191

The lower control limit = D3 * mean range = 0 * 0.5632 = 0  

B) estimate the process mean and standard deviation

estimated process mean = 50.15 = grand sample mean

standard deviation = mean range / d2  = 0.5632 / 2.326 = 0.2421

note d2 is obtained from control table

Let T:V→W be a linear transformation from a vector space V into a vector space W. Prove that the range of T is a subspace of W.

Answers

Answer:

The range of T is a subspace of W.

Step-by-step explanation:

we have T:V→W

This is a linear transformation from V to W

we are required to prove that the range of T is a subspace of W

0 is a vector in range , u and v are two vectors in range T

T = T(V) = {T(v)║v∈V}

{w∈W≡v∈V such that T(w) = V}

T(0) = T(0ⁿ)

0 is  Zero in V

0ⁿ  is zero vector in W

T(V) is not an empty subset of W

w₁, w₂   ∈ T(v)

(v₁, v₂ ∈V)

from here we have that

T(v₁) = w₁

T(v₂) = w₂

t(v₁) + t(v₂) = w₁+w₂

v₁,v₂∈V

v₁+v₂∈V

with a scalar ∝

T(∝v) = ∝T(v)

such that

T(∝v) ∈T(v)

so we have that T(v) is a subspace of W. The range of T is a subspace of W.

The state of CT claims that the average time on death row is 15 years. A random survey of 75 death row inmates revealed that the average length of time on death row is 17.8 years with a standard deviation of 5.9 years. Conduct a hypothesis to test the state of CT's claim. What type of test should be run? t-test of a mean z-test of a proportion The alternative hypothesis indicates a right-tailed test left-tailed test two-tailed test Calculate the p-value. What is the decision? We reject the claim that the average time on death row is 15 years We fail to reject the claim that the average time on death row is 15 years

Answers

Answer:

a)The calculated value t = 4.111 > 1.9925 at 5 % level of significance

Null hypothesis is rejected

The claim that the average time on death row is not 15 years

b) The p-value is 0.000101<0.05

we reject Null hypothesis

The claim that the average time on death row is not 15 years

Step-by-step explanation:

Step(i):-

Sample size 'n' =75

Mean of the sample x⁻ = 17.8

standard deviation of the sample (S) = 5.9

Mean of the Population = 15

Null hypothesis:H₀:μ = 15 years

Alternative Hypothesis :H₁:μ≠15 years

Step(ii):-

Test statistic

                         [tex]t = \frac{x^{-} -mean}{\frac{S}{\sqrt{n} } }[/tex]

                        [tex]t = \frac{x^{-} -mean}{\frac{S}{\sqrt{n} } }=\frac{17.8-15}{\frac{5.9}{\sqrt{75} } }[/tex]

                    t = 4.111

Degrees of freedom

ν = n-1 = 75-1=74

t₀.₀₂₅ = 1.9925

The calculated value t = 4.111 > 1.9925 at 5 % level of significance

Null hypothesis is rejected

The claim that the average time on death row is not 15 years

P-value:-

The p-value is 0.000101<0.05

we reject Null hypothesis

The claim that the average time on death row is not 15 years

PLEASE ANSWER FAST I WILL MARK BRAINLEIST AMD 20 POINTSBased on the figure below what is the value of X

Answers

Answer:

[tex]\boxed{9}[/tex]

Step-by-step explanation:

The two angles are complementary to each other.

That means they add up to 90 degrees.

[tex]5x+15+30=90[/tex]

[tex]5x+45=90[/tex]

[tex]5x=45[/tex]

[tex]x=9[/tex]

Answer:

x = 9

Step-by-step explanation:

So you know that the total is 90 degrees.

What you need to do is create an equation.

5x + 15 + 30 = 90

Then, solve the equation like this.

5x + 15 + 30 = 90

5x + 45 = 90

5x = 90 - 45

5x = 45

x = 45 ÷ 5

x = 9

Hope this helps! :)

Find the probability of each of the following, if Z~N(μ = 0,σ = 1).
(please round any numerical answers to 4 decimal places)
a) P(Z > -1.13) =
b) P(Z < 0.18) =
c) P(Z > 8) =
d) P(| Z | < 0.5) =

Answers

Answer: a) 0.8708, b) 5714, c) 0.000, d) 0.3830

Step-by-step explanation:

(a)

To find P(Z>-1.13):

Since Z is negative, it lies on left hand side of mid value.

Table of Area Under the Standard Normal Curve gives area = 0.3708

So,

P(Z>-1.13) = 0.5 + 0.3708 = 0.8708

(b)

To find P(Z<0.18):

Since Z is positive, it lies on right hand side of mid value.

Table of Area Under the Standard Normal Curve gives area = 0.0714

So,

P(Z<0.18) = 0.5 + 0.0714 = 0.5714

(c)

To find P(Z>8):

Since Z is positive, it lies on right hand side of mid value.

Table of Area Under the Standard Normal Curve gives area = 0.5 nearly

So,

P(Z>8) = 0.5 - 0.5 nearly = 0.0000  

(d)

To find P(| Z | < 0.5)

that is

To find P(-0.5 < Z < 0.5):

Case 1: For Z from - 0.5 to mid value:

Table of Area Under the Standard Normal Curve gives area = 0.1915

Case 2: For Z from mid value to 0.5:

Table of Area Under the Standard Normal Curve gives area = 0.1915

So,

P(| Z | < 0.5) = 2 * 0.1915 = 0.3830

The Probability can be determine using z-Table. The z- table use to determine the area under the standard normal curve for any value between the mean (zero) and any z-score.

(a) The value of [tex]P(z>-1.13)=0.8708[/tex].

(b) The value of  [tex]P(Z < 0.18) = 0.5714[/tex].

(c) The value of [tex]P(Z > 8) = 0.0000[/tex].

(d) The value of [tex]P(| Z | < 0.5) =0.3830[/tex].

Given:

The given condition is [tex]Z\sim N(\mu= 0,\sigma = 1).[/tex]

(a)

Find the value for [tex]P(Z > -1.13)[/tex].

Here Z is less than 1 that means Z is negative. So it will lies it lies on left hand side of mid value.

Refer the table of Area Under the Standard Normal Curve.

[tex]\rm Area = 0.3708[/tex].

Now,

[tex]P(Z > -1.13)=0.5 + 0.3708 = 0.8708[/tex]

Thus, the value of [tex]P(z>-1.13)=0.8708[/tex].

(b)

Find the value for [tex]P(Z < 0.18)[/tex].

Here Z is positive. So it will lies it lies on right hand side of mid value.

Refer the table of Area Under the Standard Normal Curve.

[tex]\rm Area = 0.0714[/tex].

Now,

[tex]P(Z <0.18)=0.5 + 0.0714 = 0.5714[/tex]

Thus, the value of  [tex]P(Z < 0.18) = 0.5714[/tex].

(c)

Find the value for [tex]P(Z >8)[/tex].

Here Z is positive. So it will lies it lies on right hand side of mid value.

Refer the table of Area Under the Standard Normal Curve.

[tex]\rm Area \approx 0.5[/tex].

Now,

[tex]P(Z >8)\approx0.5 - 0.5 = 0.0000[/tex]

Thus, the value of [tex]P(Z > 8) = 0.0000[/tex].

(d)

Find the value for [tex]P(|Z| <0.05)[/tex].

Here Z is mod of Z, it may be positive or negative. Consider the negative value of Z.

Refer the table of Area Under the Standard Normal Curve.

[tex]\rm Area =0.1915[/tex].

Consider the positive  value of Z.

Refer the table of Area Under the Standard Normal Curve.

[tex]\rm Area =0.1915[/tex].

Now,

[tex]P(|Z| <0.5)=2\times 0.1915 = 0.3830[/tex]

Thus, the value of [tex]P(| Z | < 0.5) =0.3830[/tex].

Learn more about z-table here:

https://brainly.com/question/16051105

Need Help with these (Giving brainiest if you can solve these)

Answers

Answer: try using sine for this equasion

Step-by-step explanation:

x(x+3)(x+3)=0 Please I NEED HELP FAST! PLLLLLLLLLLLLLLLLLLLLLLLLLLEEEEEEEEEEEEEEEEEEEEEEEEEEEEEEEEEAAAAAAAAAAAAAAAAAAAAAAAAAAAAASSSSSSSSSSSSSSSSSSSSSSSSSSEEEEEEEEEEEEEEEEEEEEEEEEEEEEEEEEEEEE!

Answers

Answer:

[tex]\boxed{x^3+6x^2+9x}[/tex]

Step-by-step explanation:

[tex]x(x+3)(x+3)[/tex]

Resolving the first parenthesis

[tex](x^2+3x) (x+3)[/tex]

Using FOIL

[tex]x^3+3x^2+3x^2+9x[/tex]

Adding like terms

[tex]x^3+6x^2+9x[/tex]

[tex]\text{If } \: a\cdot b \cdot c = 0 \text{ then } a=0 \text{ or } b =0 \text{ or } c=0 \text{ or all of them are equal to zero.}[/tex]

[tex]x(x+3)(x+3) =0[/tex]

[tex]\boxed{x_1 =0}[/tex]

[tex]x_2+3 =0[/tex]

[tex]\boxed{x_2 = -3}[/tex]

[tex]x_3+3 =0[/tex]

[tex]\boxed{x_3 = -3}[/tex]

When testing the claim that p 1p1equals=p 2p2​, a test statistic of zequals=2.04 is obtained. Find the ​p-value obtained from this test statistic.

Answers

Answer:

0.0414 with an upper tailed test

Step-by-step explanation:

Claim: P1P1 = P2P2

The above is a null hypothesis

The alternative hypothesis for a two-tailed test would be:

P1P1 \=/ P2P2

Where "\=/" represents "not equal to".

Using a z-table or z-calculator, we derive the p-value (probability value) for the z-score 2.04

With an upper tailed test, the

2 × [probability that z>2.04] = 2[0.0207] = 0.0414

This is the p-value for the test statistic.

Focus is on the alternative hypothesis.

The measure of minor arc JL is 60°. Circle M is shown. Line segments M J and M L are radii. Tangents J K and L K intersect at point K outside of the circle. Arc J L is 60 degrees. What is the measure of angle JKL? 110° 120° 130° 140°

Answers

Answer:

angle JKL =  120 degrees

Step-by-step explanation:

Since arc JL is 60 degrees, the central angle is also 60 degrees.

Point K is the intersection of tanges at J and L, therefore KJM and KLM are 90 degrees.

Consider quadrilateral JKLM whose sum of internal angles = 360.

Therefore

angle JKL + angle KLM + angle LMJ + angle MJK = 360 degrees

angle JKL + 90 + 60 + 90 = 360

angle JKL = 360 - 90 - 60 -90 = 120 degrees

Answer:

120 degrees

Step-by-step explanation:

Since arc JL is 60 degrees, the central angle is also 60 degrees.

Point K is the intersection of tanges at J and L, therefore KJM and KLM are complementary or equal 90 degrees.

look at quadrilateral JKLM whose sum of internal angles = 360.

Therefore

angle JKL plus angle KLM plus angle LMJ plus angle MJK = 360 degrees

angle JKL + 90 + 60 + 90 = 360

Question 15 of 25
What is the solution to this equation?
X + 8 = -3​

Answers

Answer:

x=-11

Step-by-step explanation:

x+8=-3

x=-3-8 :- collect like term

since we are adding two negative numbers, we will let the number be negative but add them.

x=-11

Hope it helps :)

Answer:

x=-11

Step-by-step explanation:

x+8=-3

collect like terms;

x=-3-8

x=-11

What point lies on the line described by the equation below? Y+3=2 (x-1

Answers

Answer:

[tex]\boxed{(1, -3)}[/tex]

Step-by-step explanation:

[tex]y+3=2 (x-1)[/tex]

Put equation in slope-intercept form.

[tex]y=mx+b[/tex]

[tex]y=2(x-1)-3[/tex]

[tex]y=2x-2-3[/tex]

[tex]y=2x-5[/tex]

Let x = 1

[tex]y=2(1)-5[/tex]

[tex]y=2-5[/tex]

[tex]y=-3[/tex]

The point (1, -3) lies on the line.

When randomly selecting an adult, let B represent the event of randomly selecting someone with type B blood. Write a sentence describing what the rule of complements below is telling us. P B or B = 1 Choose the correct answer below. A. It is impossible that the selected adult has type B blood or does not have type B blood. B. It is certain that the selected adult has type B blood. C. It is certain that the selected adult has type B blood or does not have type B blood. D. It is certain that the selected adult does not have type B blood.

Answers

Answer: The rule of complements is apprising us that, the person selected will.eithwr have a type B blood or will not have a type B blood

Step-by-step explanations:

Find explanations in the attachment

Consider this quote: "In a recent survey, 65 out of 100 consumers reported that they preferred plastic bags instead of paper bags for their groceries. If there is no difference in the proportions who prefer each type in the population, the chance of such extreme results in a sample of this size is about .03. Because .03 is less than .05, we can conclude that there is a statistically significant difference in preference." Give a numerical value for each of the following.

a. The p-value.
b. The level of significance, α.
c. The sample proportion.
d. The sample size.
e. The null value.

Answers

Answer:

Step-by-step explanation:

The p value (probability of obtaining results as extreme the z score if null is true) is usually the value derived to make a conclusion and in this case the p value is 0.03

The level of significance is the value usually compared with the p value which is 0.05

The sample promotion is 65 out of 100 = 65/100 = 0.65

The sample size is the total number of consumers which is 100

The null value is usually the default value. The null value would assume that there is no difference in the proportions who prefer each type in the population. There are two preferences: 100/2 = 50- 0.5 for each preference.

which of the following is equivalent to the expression below? log2-log14 A. LOG(1/7) B. LOG(-12) C. LOG 12 D. LOG 7

Answers

Answer:

The answer is option A.

Step-by-step explanation:

Using the properties of logarithms

that's

[tex] log(x) - log(y) = log( \frac{x}{y} ) [/tex]

log 2 - log 14 is

[tex] log(2) - log(14) = log( \frac{2}{14} ) [/tex]

Simplify

We have the final answer as

[tex] log( \frac{1}{7} ) [/tex]

Hope this helps you

Answer:

log ( 1/7)

Step-by-step explanation:

log2-log14

We know that log ( a/b) = log a - log b

log (2 /14)

log ( 1/7)

Find the present value of an investment that is worth $19,513.75 after earning 3% simple interest for 512 years.

Answers

Answer:

$16,750.00

Step-by-step explanation:

Simple interest:

I = Prt

Value of an investment of value P over t years at r interest rate:

F = P + Prt

F = P(1 + rt)

19,513.75 = P(1 + 0.03 * 5.5)

1.165P = 19,513.75

P = 16,750

Answer: $16,750.00

The present value of the investment was $16,750 which is worth $19,513.75 after earning 3% simple interest for 512 years.

What is the simple interest?

Simple interest is defined as interest paid on the original principal and calculated with the following formula:

S.I. = P × R × T, where P = Principal, R = Rate of Interest in % per annum, and T = Time, usually calculated as the number of years. The rate of interest is in percentage r% and is to be written as r/100

We have been given data as:

Rate of Interest (R) = 3% = 3/100 = 0.03

Time (T) = 512  years

Value of an investment of value P over t years at r interest rate:

A = P + Prt

A = P(1 + rt)

19,513.75 = P(1 + 0.03 × 5.5)

19,513.75 = 1.165P

1.165P = 19,513.75

P = 19,513.75/1.165

P = 16,750

Thus, the present value of the investment was $16,750 which is worth $19,513.75 after earning 3% simple interest for 512 years.

Learn more about the simple interest here:

brainly.com/question/22621039

#SPJ2

In a random sample of 40 refrigerators, the mean repair cost was $150. Assume the population standard deviation is $15.50. Construct a 99% confidence interval for the population mean repair cost. Then change the sample size to n = 60. Which confidence interval has the better estimate?

Answers

Answer:  ($143.69, $156.31)

Step-by-step explanation:

Confidence interval to estimate  population mean :

[tex]\overline{x}\ \pm z\dfrac{\sigma}{\sqrt{n}}[/tex]

, where [tex]\sigma[/tex] = population standard deviation

n= sample size

[tex]\overline{x}=[/tex] Sample mean

z= critical value.

As per given,

n= 40

[tex]\sigma[/tex] = $15.50

[tex]\overline{x}=[/tex] $150

Critical value for 99% confidence level = 2.576

Then, 99% confidence interval for the population mean:

[tex]150\pm(2.576)\dfrac{15.50}{\sqrt{40}}\\\\\Rightarrow\ 150\pm6.31 \ \ (approx)\\\\\Rightarrow(150-6.31,150+6.31)=(143.69,156.31)[/tex]

Hence, the required confidence interval : ($143.69, $156.31)

What is the inverse of the logarithmic function

f(x) = log2x?


f –1(x) = x2


f –1(x) = 2x


f –1(x) = logx2


f –1(x) = StartFraction 1 Over log Subscript 2 Baseline x EndFraction

Answers

Answer:

B. edge 2021

B. is correct for the next one too.

Step-by-step explanation:

B. is the correct answer for the first one
B. is also the correct answer for the second one

WHY IS THERE ANY HELP? PLEASE Solve the system of equations by using the substitution method. [tex]\left \{ {{x+y=6} \atop {x=2y}} \right.[/tex] Is there a solution, no solution, or infinite number? If there's a solution, what's the ordered pair?

Answers

Answer:

There is a solution. The ordered pair is (4, 2).

Step-by-step explanation:

Solve the system of equations by using the substitution method.

[tex]x+y=6\\x=2y[/tex]

Substitute x as 2y in the first equation and solve for y.

[tex]2y+y=6\\ 3y=6\\(3y)/3=6/3\\y=2[/tex]

Substitute y as 2 in the second equation and solve for x.

[tex]x=2(2)\\x=4[/tex]

Using Pascal’s Theorem, expand the expression 〖(2x-y)〗^3

Answers

Answer:

(2x - y)³ = 8x³ - 12x²y + 6xy² - y³

Step-by-step explanation:

Pascal's Theorem uses a set of already known and easily obtainable numbers in the expansion of expressions. The numbers serve as the coefficients of the terms in the expanded expression.

For the expansion of

(a + b)ⁿ

As long as n is positive real integer, we can obtain the coefficients of the terms of the expansion using the Pascal's triangle.

The coefficient of terms are obtained starting from 1 for n = 0.

- For the next coefficients of terms are 1, 1 for n = 1.

- For n = 2, it is 1, 2, 1

- For n = 3, it is 1, 3, 3, 1

The next terms are obtained from the previous one by writing 1 and summing the terms one by one and ending with 1.

So, for n = 4, we have 1, 1+3, 3+3, 3+1, 1 = 1, 4, 6, 4, 1.

The Pascal's triangle is

1

1 1

1 2 1

1 3 3 1

1 4 6 4 1

1 5 10 10 5 1

1 6 15 20 15 6 1

1 7 21 35 35 21 7 1

1 8 28 56 70 56 28 8 1

1 9 36 84 126 126 84 36 9 1

The terms can also be obtained from using the binomial theorem and writing the terms from ⁿC₀ all through to ⁿCₙ

So, for n = 3, the coefficients are 1, 3, 3, 1

Then the terms are written such that the sum of the powers of the terms is 3 with one of the terms having the powers reducing from n all through to 0, and the other having its powers go from 0 all through to n

So,

(2x - y)³ = [(1)(2x)³(-y)⁰] + [(3)(2x)²(-y)¹] + [(3)(2x)¹(-y)²] + [(1)(2x)⁰(-y)³]

= (1×8x³×1) + (3×4x²×-y) + (3×2x×y²) + (1×1×-y³)

= 8x³ - 12x²y + 6xy² - y³

Hope this Helps!!!

For the functions f(x)=8 x 2 +7x and g(x)= x 2 +2x , find (f+g)(x) and (f+g)(3)

Answers

Answer:

(f+g)(x)= 9x² + 9x

(f+g)(3) = 108

Step-by-step explanation:

f(x)=8x² +7x

g(x)= x² +2x

(f+g)(x) = f(x) + g(x) = 8x² +7x +x² +2x = 9x² + 9x

(f+g)(x)= 9x² + 9x

(f+g)(3)= 9*3² + 9*3 = 108

what is the length of bc in the right triangle below? ​

Answers

Answer: A) 15

Step-by-step explanation:

Because of Pythagorean Theorem, 9^2+12^2=BC^2.  Thus, 81+144=BC^2.  Thus, 225=BC^2.  Thus, 15=BC.

Hope it helps, and ask if you want further clarification <3

Section 8
Find the mean of these numbers:
24 18
37
82 17
26​

Answers

Answer:

[tex]\boxed{Mean = 34.33}[/tex]

Step-by-step explanation:

Mean = Sum of Observations / No. Of Observations

Mean = (24+18+37+82+17+26)/6

Mean = 206 / 6

Mean = 34.33

Find the length of AG

Answers

Answer:

[tex]AG=22[/tex]

Step-by-step explanation:

Follow the next steps:

[tex]\frac{A-B}{A-E} =\frac{B-C}{E-F} =\frac{C-D}{F-G} =\frac{A-C}{A-F} =\frac{B-D}{E-G} =\frac{A-D}{A-G}[/tex]

Let:

[tex]\frac{A-B}{A-E} =\frac{B-C}{E-F}\\ \\\frac{4}{A-E} =\frac{5}{10x}\\ \\Solving\hspace{3}for\hspace{3}A-E\\\\A-E=8x[/tex]

Now:

[tex]\frac{C-D}{F-G} =\frac{A-C}{A-F} \\\\\frac{2}{F-G} =\frac{9}{18x} \\\\Solving\hspace{3}for\hspace{3}F-G\\\\F-G=4x[/tex]

Hence:

[tex]A-G=(A-E)+(E-F)+(F-G)=22x[/tex]

Finally:

[tex]\frac{B-D}{E-G} =\frac{A-D}{A-G}\\\\\frac{A-D}{B-D} =\frac{A-G}{E-G}\\[/tex]

[tex]\frac{11}{7} =\frac{22x}{14x} \\\\\frac{11x^{2} }{7} -\frac{11}{7} =0\\\\[/tex]

Hence:

[tex]x=1\\x=-1[/tex]

Since it would be absurd for [tex]x=-1[/tex], the real solution is [tex]x=1[/tex]

Therefore:

[tex]AG=22[/tex]

when Charles eats Oreos , he likes to dunk 2 out of every 5 cookies in a cold glass of milk. if he eats a total of 15 Oreos , how many will he dunk ? how many will ge eat without dunking?

Answers

Answer: 6 with milk, 9 without

Step-by-step explanation:

2/5 of the cookies he eats are dunked.  Thus, simply do 2/5, or .4*15 to get that 6 cookies are dunked, and 15-6 to get that 9 cookies are not dunked.

Hope it helps <3

RVLC2019] IC/Off

In AMNO, m = 20, n = 14, and mZM = 51°. How many distinct triangles can be formed given these measurements?

O There are no triangles possible.

VX

O There is only one distinct triangle possible, with m N= 33º.

O There is only one distinct triangle possible, with mZN 147º.

O There are two distinct triangles possible, with m2N 33° or mZN-147º.

Done

) Intro

DO

Answers

The answer would have to be 33

There is only one distinct triangle possible, with m N= 33º. Therefore, option B is the correct answer.

What is sine rule?

Law of Sines In trigonometry, the law of sines, sine law, sine formula, or sine rule is an equation relating the lengths of the sides of any triangle to the sines of its angles.

The formula for sine rule is sinA/a=sinB/b=sinC/c

Given that, in ΔMNO, m = 20, n = 14, and m∠M = 51°.

Now, sin51°/20=sinN/14

0.7771/20=sinN/14

0.038855=sinN/14

sinN=14×0.038855

sinN=0.54397

N=33°

Therefore, option B is the correct answer.

Learn more about the sine rule here:

https://brainly.com/question/22288720.

#SPJ7

Which describes the graph in words?
A. All numbers less than -10 and less than or equal to 8.
B. All numbers greater than -10 and less than 8
C. All numbers greater than or equal to -10 and less than or equal to 8
D. All numbers greater than -10 and less than or equal to 8.

Answers

D. All numbers greater than -10 and less than or equal to 8

Complete the table.PLSSS HELP ILL GIVE BRAINLIEST.PLS PLS PLS PLS

Answers

Answer:

0, 22, 44, 66

Step-by-step explanation:

Given the equation for the model, [tex] d = 11t [/tex] , you can complete the table above by simply plugging in each value of "t" has given in the table to solve for "d".

*When t (seconds) = 0, distance (feet) would be:

[tex] d = 11(0) [/tex]

[tex] d = 0 [/tex]

*When t (seconds) = 2, distance (feet) would be:

[tex] d = 11(2) [/tex]

[tex] d = 22 [/tex]

*When t (seconds) = 4, distance (feet) would be:

[tex] d = 11(4) [/tex]

[tex] d = 44 [/tex]

*When t (seconds) = 6, distance (feet) would be:

[tex] d = 11(6) [/tex]

[tex] d = 66 [/tex]

expand(x+y2)2 plzzzzzzzzzzzzzzzz​

Answers

Answer:

[tex](x + {y}^{2}) = {x}^{2} + 2x {y}^{2} + {y}^{4} [/tex]

Hope it helps!!❤❤Please mark me as the brainliest!!!

Thanks!!!!

Other Questions
(20 POINTS!!!) Richie and his brother have a paper route. Together they can deliver all of the papers in 40 minutes, and Richie can do it alone in 90 minutes. Richies brother slept in one day, leaving Richie to deliver alone for 30 minutes. How long must the two of them work together to finish delivering the newspapers? Find the measure of b. What is the circumference of a circle with a diameter of 100m. A 100m B 157m C 300 m D 314m A number is 30% of 20% of the number x. Given the coordinates for the function below, which of the following arecoordinates for its inverse?Gallons Cost, inof Gas Dollars12515201.252.506.2518.7525.00 Which system of equations represents the matrix shown below? From the set {20, 30, 35}, use substitution to determine which value of x makes the inequality true. x - 5 > 25 A. 30 B. 20 C. none of these D. 35 The inequality x < 9 or x 14 can be used to represent the hourly wage, x, of each employee at a store. Which are possible values for x? Select two options. $8 $9 $11 $13 $14 sin theta = x , sec theta =y . find cot theta pls answer fast i need to verify my answer . you can directly write the answer no issues solve the inquality 1/2* I NEED HELP PLEASE ANSWER QUICK (SAT Prep) In the given figure, a b. Find the value of z. A. 50 B. 90 C. 45 D. 75 Pause at line 55. Why areByres's drawings from the1780s so helpful to restorerstoday?Please help I need to graduate!! during which stage of a collaborative discussion should a person acknowledge key points others' statements and ask necessary clarifying questions Tasha wants to find out if she is going to pass her test. She decides that she willsimulate her test by flipping a coin. This meansa) There is a 50-50 chance she will pass her test.b) All of the questions are multiple choice.c) She hasn't studied.d) There is a 20% chance she will pass. Question 5 of 15Water turns from a gas to a liquid in a process calledA. evaporationB. sublimationc. transpositionD. condensationSUBMIT Three books are at rest, in equilibrium, on a horizontal table asshown. The weight of each book, which is equal to the forcegravity exerts in the downward direction, is given. What is thenet force on the middle book? Etsuro is a management consultant. Baker Corp. asks him to evaluate their company, and he finds that the difference between the cost of producing the firm's products and the value of those products is extremely narrow. What should Etsuro suggest that Baker Corp. management do? Find a way to pass on as much profit as possible to suppliers and customers. Find a way to widen the gap between cost and value. Shore up the company's strong position by erecting entry barriers. Encourage customers to buy complements to their products. Offshore oil-drilling operations entail an unavoidable risk of an oil spill, but importing oil on tankers presently entails an even greater such risk per barrel of oil. Therefore, if we are to reduce the risk of an oil spill without curtailing our use of oil, we must invest more in offshore operations and import less oil on tankers. Which of the following, if true, most seriously weakens the argument above. A) Tankers can easily be redesigned so that their use entails less risk of an oil spill. B) Oil spills caused by tankers have generally been more serious than those caused by offshore operations. C) The impact of offshore operations on the environments can be controlled by careful management. D) Offshore operations usually damage the ocean floor, but tankers rarely cause such damage. E) Importing oil on tankers is currently less expensive than drilling for it offshores. A lichen is an organism that structurally appears to be a single organism. But a lichen is actually two different organismsa fungus and green algaeliving together as one organism. The fungal partner derives its nutrition from the photosynthesizing algae. How does a lichen differ in its photosynthetic activity from Elysia chlorotica, the sea slug thats considered to be a photosynthesizing animal? A. In lichens, the fungi photosynthesize on their own, while Elysia chlorotica forms a relationship with a photosynthesizing plant. B. In lichens, the relationship with the algae lasts throughout the life cycle, but in Elysia chlorotica, the relationship occurs only during the immature juvenile stage of the slugs life cycle. C. Lichens can photosynthesize only because of the living algal partner, while Elysia chlorotica incorporates chloroplasts from algae into its cells. D. In lichens, the association between the fungi and algae is permanent, while Elysia chlorotica associates with a photosynthesizing organism only when it requires food. PLEASE HELP The probability distribution for arandom variable x is given in the table.